Montrer que \[\forall x \in ]0, 1[,\quad x^x(1-x)^{1-x} \geqslant\dfrac{1}{2}.\]


Barre utilisateur

[ID: 927] [Date de publication: 18 janvier 2021 15:44] [Catégorie(s): Convexité ] [ Nombre commentaires: 1] [nombre d'éditeurs: 1 ] [Editeur(s): Emmanuel Vieillard-Baron ] [nombre d'auteurs: 3 ] [Auteur(s): Alain Soyeur François Capaces Emmanuel Vieillard-Baron ]




Solution(s)

Solution(s)

Exercice 357
Par Alain Soyeur François Capaces Emmanuel Vieillard-Baron le 18 janvier 2021 15:44

Comme la fonction \(\ln\) est concave sur \(]0, 1[\), \[\forall (a, b)\in ]0, 1[^2, \forall \lambda \in [0, 1], \quad \ln(\lambda a + (1-\lambda )b ) \geqslant\lambda \ln a + (1-\lambda) \ln b.\] Soit \(x\in ]0,1[\). En posant \(a=x\in]0, 1[\), \(b=1-x\in]0, 1[\) et \(\lambda = x\in]0, 1[\) , on trouve que \[x\ln x + (1-x)\ln(1-x) \leqslant\ln( x^2 + (1-x)^2)\] En étudiant la fonction \(\varphi: \left\{ \begin{array}{ccl} \mathbb{R} & \longrightarrow & \mathbb{R} \\ x & \longmapsto & x^2 + (1-x)^2 \end{array} \right.\), on montre qu’elle admet un minimum en \(\dfrac{1}{2}\) qui vaut \(\dfrac{1}{2}\). Par conséquent, \[x\ln x + (1-x) \ln(1-x) \geqslant\ln(\dfrac{1}{2}).\] On applique alors l’exponentielle à notre inégalité et comme cette fonction est croissante, on obtient le résultat de l’énoncé.


Documents à télécharger